10
$\begingroup$

$\DeclareMathOperator{\crit}{\operatorname{crit}}$A rank-into-rank embedding is a non-trivial elementary embedding from a rank initial segment of $V$ into itself: $j:V_\delta\prec V_\delta$. Define the critical sequence of such an embedding by setting $\kappa_0=\crit(j)$ (the first ordinal moved by $j$) and $\kappa_{n+1}=j(\kappa_n)$. Let $\lambda=\crit^\omega(j)=\sup_{n<\omega} \langle \kappa_n\rangle$. It is straightforward to see that $\lambda$ is a strong limit cardinal of countable cofinality.

By a theorem of Kunen, if such an embedding can exist, then $\delta$ must be the ordinal $\lambda$ or $\lambda+1$.

It is not hard to see that $\crit(j)$ must be measurable. In fact, for any $n$, $\crit(j)$ is also $n$-huge as witnessed by the ultrafilter $$U=\{X\subseteq\mathcal{P}(\kappa_n): j"\kappa_n\in j(X)\}.$$ Further, if we let $j^n$ denote $j$ composed with itself $n$ times, then $$V_\lambda\models ``\lambda\text{ is supercompact"}.$$ To see this, suppose $\crit(j)\leq \theta <\kappa_n$, then $$U=\{X\subseteq\mathcal{P}_{\crit(j)}(\theta): j^n"\theta\in j^n(X)\}$$ winesses the $\theta$-compactness of $\crit(j)$ (in $V_\lambda$).

For the last claim, it is enough that $\crit(j)$ is $<\lambda$-supercompact, i.e. not fully supercompact in $V$. In this case, however, $\crit(j)$ could be fully supercompact.

But extendible cardinals are not characterized by the presence of ultrafilters and this motivates my question here.

Question: Can the critical point of a rank-into-rank embedding be extendible?

It may not make sense (I think) to ask for full extendibility of $\crit(j)$: Suppose otherwise that $\crit(j)$ is fully extendible. Let $k$ witness the $\theta$-extendibility of $\crit(j)$ for some $\theta>\crit^\omega(j)$. Then we have $$V_{\crit(j)}\prec V_{\crit^\omega(j)}\prec V_\theta.$$
This looks suspiciously like Woodin's Enormous Cardinal (though his notion is defined in the context of just ZF). See http://logic.harvard.edu/EFI_Woodin_talk.pdf, slide 20. Thus I'm not sure that $\crit(j)$ can be fully extendible.

Question: Assume $j$ is a rank-into-rank embedding and let $\lambda=\crit^\omega(j)$. Can $\crit(j)$ be $<\lambda$-extendible?

Edit: I should point out (reminded by Carlo Von Shnitzel's comments below) that there is a sort of local intertwining of supercompact cardinals and extendible cardinals that may be relevant. See Kanamori's book, p.316-318.

Also, there may be some subtlety here concerning $\Sigma_k$ correctness. Suppose $$j:V_\lambda\prec V_\lambda.$$ I think assuming $V_\lambda\prec_3 V$ (or even $V_\lambda\prec_2 V$) is a strictly stronger assumption. If $\crit(j)$ were extendible, then $V_{\crit(j)}\prec_3 V$. But the embedding assumption also gives us that $V_{\crit^\omega(j)}\prec_3 V$, even though $\crit^\omega(j)=\lambda$ is not itself an extendible cardinal. Similarly if we assume $\crit(j)$ is actually supercompact.

$\endgroup$
3
  • 1
    $\begingroup$ If $\kappa=crit(j)$ is $\theta$-supercompact for some $\theta$ and if we let $j:V \to M$ witness this $\theta$- supercompactness then since $j|V_{\alpha}: V_{\alpha} \to j(V_{\alpha})= M_{j(\alpha)}$ is bounded by $\theta$ so it is in $M$, by the supercompactness, we get that $\kappa$ is $\alpha$-extendible for any $\alpha$ such that $\beth_{\alpha} \leq \theta$. We can get the appropriate supercompactness from embeddings $j:V_{\theta} \to V_{\theta}$, say by $X \in \mu \leftrightarrow j"\delta \in j(X)$ with $X \subset P_{\kappa}(\delta)$ if... $\endgroup$ Nov 4, 2013 at 7:04
  • $\begingroup$ ...$j(\kappa)>\delta$ and if $P_{\kappa}(\delta) \subset V_{\theta}$. I'm not sure about it and in any case you are asking about the sup of the critical sequence. $\endgroup$ Nov 4, 2013 at 7:04
  • $\begingroup$ This wrong. We can't get $M_{j(\alpha)}\subseteq M$ from $M^\alpha\subseteq M$; we can only get it from $M^{j(\alpha)}\subseteq M$. For this reason, the least supercompact cardinal is not $1$-extendible, but is a stationary limit of cardinals that $1$-extendible. $\endgroup$
    – Master
    Oct 7, 2019 at 2:44

1 Answer 1

3
$\begingroup$

Theorem: If $\kappa$ is the critical point of $j\colon V_\lambda\prec V_\lambda$, then $\kappa$ is $\lambda$-weakly extendible. Furthermore, if $\kappa$ is the critical point of $j\colon V_\lambda\prec V_\lambda$, then $\kappa$ is $\lt\lambda$-strongly extendible.

Proof. First off $\kappa+\lambda=\lambda$, as $\kappa_n$ is a cardinal for each $n$ ($\kappa_0=\kappa$), and therefore $\lambda$ is a cardinal $>\kappa$. By definition, there is an elementary embedding $j\colon V_\lambda\prec V_\lambda$ with critical point $\kappa$. Similarly, for each $\alpha\lt\lambda$ such that $\alpha\lt j^n(\kappa)$, $\kappa$ is strongly $\alpha$-extendible as witnseesed by $j^{(n)}\restriction V_\alpha: V_\alpha\prec V_{j^n(\alpha)}$.■

Theorem: If $\kappa$ is the critical point of $j\colon V_\lambda\prec V_\lambda$ and $\kappa\in C^{(2)}$, then $\kappa\gt$ the least rank-into-rank cardinal. Furthermore, if $\kappa$ is $\lambda$-strongly extendible, then $\lambda>$ the least rank-into-rank cardinal.

Proof. The statements “$\lambda$ is rank-into-rank” and “there exists a rank-into-rank embedding” are both $\Sigma_2$. And so if $V_\kappa\prec_{\Sigma_2} V$, then $V_\kappa\vDash\text{There is a rank-into-rank embedding}$ and if $V_\kappa\vDash\lambda_0\text{ is rank-into-rank}$, then $\lambda_0$ is rank-into-rank. For the second part, let $k: V_\lambda\prec V_{k(\lambda)}$ witness $\lambda$-strong extendibility. Then $k(\kappa)+1\lt k(\lambda)$ and so $V_{k(\kappa)+1}\subseteq V_{k(\lambda)}$ and therefore $k(\kappa)$ is inaccessible. Therefore, $V_{k(\kappa)}\vDash\text{There is a rank-into-rank embedding}$, and so $V_\kappa\vDash\text{There is a rank-into-rank embedding}$ (As $V_\kappa\prec V_{k(\kappa)}$), and if $V_\kappa\vDash\lambda_0\text{ is rank-into-rank}$, then $\lambda_0$ is rank-into-rank, because $\Sigma_2$-formulas are upward absolute in $V_\kappa$ for inaccessible $\kappa$.■

Note then that the consistency strength of “$\kappa$ is the critical point of $j\colon V_\lambda\prec V_\lambda$, and $V_\kappa\prec_{\Sigma_2} V$” is therefore greater than I3. Furthermore, the consistency strength of “$\kappa$ is the critical point of $j\colon V_\lambda\prec V_\lambda$, and $\kappa$ is $\lambda$-strongly extendible” is therefore greater than I3.

Theorem: If $\kappa$ is I2, then the cardinals which are I3 and extendible in $V_\kappa$, are stationary in $\kappa$.

Proof. Let $X$ be the set of cardinals which are I3 in $V_\lambda$ as witnessed by some $j\subseteq V_\lambda$. Then, if $\alpha\in X$, $\alpha$ is I3, because $V_\alpha^{V_\beta}=V_\alpha$ whenever $\alpha\lt\beta$. Therefore $\kappa\in j(X)$, because $\Sigma_2^1$-properties are prserved, and so the cardinals $Y\in D$, where $Y$ is the set of cardinals which are I3 and $D$ the measure generated by $j$. Similarly, $\kappa$ is extendible in $V_\lambda$ and so $V_{j(\kappa)}$ and so $Z\in D$, where $Z$ is the set of cardinals extendible in $V_\kappa$. Therefore $Y\cap Z\in D$, so that $Y\cap Z$ is stationary, because every club set $C$ has $j(C)\cap\kappa=C$ and so $\kappa\in j(C)$.■

References: Upward reflection of rank-into-rank cardinals

$\endgroup$
1
  • $\begingroup$ I do not understand your first argument:$j^n$ has critical point $\kappa_n$ not $\kappa$. How can you then conclude that $j^n$ witnesses that $\kappa$ is $<\lambda$-extendible? $\endgroup$
    – Ergonvi
    Jun 22, 2022 at 22:24

Your Answer

By clicking “Post Your Answer”, you agree to our terms of service and acknowledge you have read our privacy policy.

Not the answer you're looking for? Browse other questions tagged or ask your own question.